Based on the image below, if you know that , find the following:

a) sin A
b) cos A
c) tan A
d) tan B

Based On The Image Below, If You Know That , Find The Following: A) Sin A B) Cos A C) Tan A D) Tan B

Answers

Answer 1

Answer:

A. Sine A = 4/5

B. Cos A = 3/5

C. Tan A = 4/3

D. Tan B = 3/4

Step-by-step explanation:

We'll begin by calculating the value of b in the attached photo.

This can be obtained as follow:

Cos B = 4/5

Cos B = Adjacent /Hypothenus

Adjacent = 4

Hypothenus = 5

Using pythagoras theory, the value of b can be obtained as follow:

b² = 5² – 4²

b² = 25 – 16

b² = 9

Take the square root of both side.

b = √9

b = 3

A. Determination of Sine A

Sine A =?

Opposite = 4

Hypothenus = 5

Sine A = Opposite /Hypothenus

Sine A = 4/5

B. Determination of Cos A

Cos A =?

Adjacent = 3

Hypothenus = 5

Cos A = Adjacent /Hypothenus

Cos A = 3/5

C. Determination of Tan A.

Tan A =?

Opposite = 4

Adjacent = 3

Tan A = Opposite /Adjacent

Tan A = 4/3

D. Determination of Tan B

Tan B =?

Opposite = 3

Adjacent = 4

Tan B = Opposite /Adjacent

Tan B = 3/4

Based On The Image Below, If You Know That , Find The Following: A) Sin A B) Cos A C) Tan A D) Tan B

Related Questions

20 PTS PLEASE HELP!!!!

Select the correct answer from each drop-down menu.

The function below describes the number of students who enrolled at a university, where f(t) represents the number of students and t represents the time in years.


Initially, (1.03, 3, 19,055, 18,500) students enroll at the university. Every,(1years, t years, 2years, 3years) the number of students who enroll at the university increases by a factor of (1.03, 3, 19,055, 18,500).

Answers

Answer:

Initially 18,500 students

Every 1 year

increase by a factor 1.03

Step-by-step explanation:

The missing information is selected from the given options from the drop down menu. The correct answers are : Initially 18,500 students enroll at the university. Every 1 years the number of students who enroll at the university increases by a factor 1.03.

F(t) = 18,500 * (1.03)^t

April typed a 5 page report in 50 mintues. Each page had 500 words at what rate is April typing

Answers

Answer:

Amy types at a rate of 50 words per minute

Step-by-step explanation:

In this question, we are interested in calculating the rate at which April is typing.

From the question, we can deduce that she typed a 5 page report, with each page having a total of 500 words.

Now, if each page has 500 words, the total number of words in all of the pages will be 5 * 500 = 2,500 words

Now, from here, we can see that 2,500 words were typed in 50 minutes.

The number of words per minute will be ;

Total number of words/Time taken = 2500 words/50 minutes

That will give a value of 50 words per minute

Can someone plz help me ASAP!!!!!!!!

Answers

Answer:

A) The number halfway between -2 and 6 is 2.

B) -10 is halfway between -18 and 8

Find the total surface area.

Answers

Answer:

143.4 mi²

Step-by-step explanation:

Top: 8x6=48

Bottom: 3x8=24

Sides: 3x8=24 and 24

Trapezoids sides: (6+3)/2*2.6=4.5*2.6=11.7 and 11.7

TOTAL: 48+24+24+24+11.7+11.7= 143.4 mi²

G(x)= -\dfrac{x^2}{4} + 7g(x)=− 4 x 2 ​ +7g, left parenthesis, x, right parenthesis, equals, minus, start fraction, x, squared, divided by, 4, end fraction, plus, 7 What is the average rate of change of ggg over the interval [-2,4][−2,4]open bracket, minus, 2, comma, 4, close bracket?

Answers

Answer:

-1/2

Step-by-step explanation:

Given the function [tex]G(x)= -\dfrac{x^2}{4} + 7[/tex], the average rate of change of g(x) over the interval [-2,4], is expressed as shown below;

Rate of change of the function is expressed as g(b)-g(a)/b-a

where a - -2 and b = 4

[tex]G(4)= -\dfrac{4^2}{4} + 7\\G(4)= -\dfrac{16}{4} + 7\\G(4)= -4 + 7\\G(4) = 3\\[/tex]

[tex]G(-2) = -\dfrac{(-2)^2}{4} + 7\\G(-2)= -\dfrac{4}{4} + 7\\G(-2)= -1 + 7\\G(-2)= 6[/tex]

average rate of change of g(x) over the interval [-2,4] will be;

[tex]g'(x) = \frac{g(4)-g(-2)}{4-(-2)}\\ g'(x) = \frac{3-6}{6}\\\\g'(x) = -3/6\\g'(x) = -1/2[/tex]

Find the product of 0.3×0.23

Answers

Answer:

0.069

Step-by-step explanation:

0.3*0.23=0.069

if your ans is correct i will choose you as a brainlist when the number of student of a school was increased by 30% it became 455. Find the previous number student.

Answers

Step-by-step explanation:

find 30% of 455

which is = 136.5

then subtract 136.5 from the original number(455)

455 - 136.5

=318.5 student

Previous number student was 350

SIMPLIFY.

(5c^2 + c) - (3c^2 + 11c)

Answers

Answer:2 c^2 - 10c

Step-by-step explanation:

Multiply. (2x - 3)(x + 4) a 2x² + 11x - 12 b 2x² + 5x - 12 c 2x² + 11x - 7 d 2x² + 3x - 7

Answers

Answer:

2x^2 +5x-12

Step-by-step explanation:

(2x - 3)(x + 4)

FOIL

first 2x*x = 2x^2

outer  2x*4 = 8x

inner  -3x

last -3*4 = -12

Add these together

2x^2 +8x-3x-12

Combine like terms

2x^2 +5x-12

The drama club is selling tickets to its play. An adult ticket costs $15 and a student ticket costs $11. The auditorium will seat 300 ticket-holders. The drama club wants to collect at least $3630 from ticket sales.

Answers

Answer:

83 adult tickets and 217 student tickets.

Step-by-step explanation:

Let number of adult tickets sold = [tex]x[/tex]

Given that total number of tickets = 300

So, number of student tickets = 300 - [tex]x[/tex]

Cost of adult ticket = $15

Cost of student ticket = $11

Total collection from adult tickets = $[tex]15x[/tex]

Total collection from student tickets =  [tex](300-x)\times 11 = 3300-11x[/tex]

Given that overall collection = $3630

[tex]15x+(3300-11x) = 3630\\\Rightarrow 15x-11x=3630-3300\\\Rightarrow 4x = 330\\\Rightarrow x = 82.5[/tex]

So, for atleast $3630 collection, there should be 83 adult tickets and (300-83 = 217 student tickets.

Now , collection = $3632

A shell of mass 8.0-kg leaves the muzzle of a cannon with a horizontal velocity of 600 m/s. Find the recoil velocity of the cannon, if its mass is 500kg.

Answers

Answer:

velocity of recoil velocity of cannon  is -9.6 m/sec

Step-by-step explanation:

according to law of conservation of momentum

total momentum of isolated system of body remains constant.

momentum  = mass of body* velocity of body.

__________________________________

in the problem the system is

shell + cannon

momentum of shell = 8*600 = 4800 Kg-m/sec

let the velocity of cannon be x m/sec

momentum of cannon = 500*x = 500x Kg-m/sec

initially the system of body is in rest (before the shell is fired) hence, total momentum of the system i is 0

applying  conservation of momentum

total momentum before shell fired = total momentum after the shell is fired

0 = momentum of shell + momentum of cannon

4800 + 500x = 0

x = -4800/500 = -9.6

Thus, velocity of recoil velocity of cannon  is -9.6 m/sec

here negative sign implies that direction of velocity of cannon is opposite to that of velocity of shell.

6 points are place on the line a, 4 points are placed on the line b. How many triangles is it possible to form such that their verticies will be the given points, if a ∥b?

Answers

Answer: 96

Step-by-step explanation:

Ok, lines a and b are parallel.

We can separate this problem in two cases:

Case 1: 2 vertex in line a, and one vertex in line b.

Here we use the relation:

"In a group of N elements, the total combinations of sets of K elements is given by"

[tex]C = \frac{N!}{(N - K)!*K!}[/tex]

Here, the total number of points in the line is N, and K is the ones that we select to make the vertices of the triangle.

Then if we have two vertices in line a, we have:

N = 6, K = 2

[tex]C = \frac{6!}{4!*2!} = \frac{6*5}{2} = 3*5 = 15[/tex]

And the other vertex can be on any of the four points on the line b, so the total number of triangles is:

C = 15*4 = 60.

But we still have the case 2, where we have 2 vertices on line b, and one on line a.

First, the combination for the two vertices in line b is:

We use N = 4 and K = 2.

[tex]C = \frac{4!}{2!*2!} = \frac{4*3}{2} = 6[/tex]

And the other vertice of the triangle can be on any of the 6 points in line a, so the total number of triangles that we can make in this case is:

C = 6*6 = 36

Then, putting together the two cases, we have a total of:

60 + 36 = 96 different triangles

Is △FHK similar to △GHJ? If so, which postulate or theorem proves these two triangles are similar? ​△FHK​ ​ is similar to ​ ​△GHJ​ ​ by the ​ ​SSS Similarity Theorem​. ​△FHK​ ​ is similar to ​ ​△GHJ​ ​ by the ​SSA Similarity Theorem​. ​△FHK​ ​ is similar to ​ ​△GHJ​ ​ by the ​ ​ ​SAS Similarity Theorem. ​△FHK​ ​ is not similar to ​ ​△GHJ​​.

Answers

Answer:

ΔFHK and ΔGHJ are the similar triangles by SAS similarity theorem.

Step-by-step explanation:

Picture for the given question is missing; find the picture attached.

If [tex]\frac{\text{FH}}{\text{GH}}=\frac{\text{HK}}{\text{HJ}}[/tex] and ∠H ≅ ∠H

Then ΔFHK ~ ΔGHJ

[tex]\frac{\text{FH}}{\text{GH}}=\frac{\text{HK}}{\text{HJ}}[/tex]

[tex]\frac{(12+10)}{10}=\frac{(15+18)}{15}[/tex]

[tex]\frac{22}{10}=\frac{33}{15}[/tex]

[tex]\frac{11}{5}=\frac{11}{5}[/tex]

Since, [tex]\frac{\text{FH}}{\text{GH}}=\frac{\text{HK}}{\text{HJ}}[/tex] and ∠H ≅ ∠H [By reflexive property]

Therefore, ΔFHK and ΔGHJ are the similar triangles by SAS similarity theorem.

Option (3) will be the answer.

Answer:

ΔFHK and ΔGHJ are similar triangles by the SAS similarity theorem.

Step-by-step explanation:

Verified correct with test results.

In politics, marketing, etc. We often want to estimate a percentage or proportion p. One calculation in statistical polling is the margin of error - the largest (reasonble) error that the poll could have. For example, a poll result of 72% with a margin of error of 4% indicates that p is most likely to be between 68% and 76% (72% minus 4% to 72% plus 4%). In a (made-up) poll, the proportion of people who like dark chocolate more than milk chocolate was 32% with a margin of error of 2.2%. Describe the conclusion about p using an absolute value inequality.

Answers

Answer: |p-72% |≤ 4%

Step-by-step explanation:

Let p be the population proportion.

The absolute inequality about p using an absolute value inequality.:

[tex]|p-\hat{p}| \leq E[/tex] , where E = margin of error, [tex]\hat{p}[/tex] = sample proportion

Given:  A poll result of 72% with a margin of error of 4% indicates that p is most likely to be between 68% and 76% .

|p-72% |≤ 4%

⇒    72% - 4% ≤ p ≤ 72% +4%

⇒  68%  ≤ p ≤  76%.

i.e. p is most likely to be between 68% and 76% (.

The conclusion about p using an absolute value inequality is in the range of 29.8% to 34.2%.

What is absolute value inequality?

An expression using absolute functions and inequality signs is known as an absolute value inequality.

We know that the absolute value inequality about p using an absolute value inequality is written as,

[tex]|p-\hat p| \leq E[/tex]

where E is the margin of error and [tex]\hat p[/tex] is the sample proportion.

Now, it is given that the poll result of 72% with a margin of error of 4% indicates that p is most likely to be between 68% and 76%. Therefore, p can be written as,

[tex]|p-0.72|\leq 0.04\\\\(0.72-0.04)\leq p \leq (0.72+0.04)\\\\0.68 \leq p\leq 0.76[/tex]

Thus, the p is most likely to be between the range of 68% to 76%.

Similarly, the proportion of people who like dark chocolate more than milk chocolate was 32% with a margin of error of 2.2%. Therefore, p can be written as,

[tex]|p-\hat p|\leq E\\\\|p-0.32|\leq 0.022\\\\(0.32-0.022)\leq p \leq (0.32+0.022)\\\\0.298\leq p\leq 0.342[/tex]

Thus, the p is most likely to be between the range of 29.8% to 34.2%.

Hence, the conclusion about p using an absolute value inequality is in the range of 29.8% to 34.2%.

Learn more about Absolute Value Inequality:

https://brainly.com/question/4688732

Can anyone tell me the answer of the question attached below??

Answers

Answer:  AE = 5

Step-by-step explanation:

I sketched the triangle based on the information provided.

since ∠A = 90° and is divided into three equal angles, then ∠BAD, ∠DAE, and ∠CAE = 30°

Since AB = 5 and BC = 10, then ΔCAB is a 30°-60°-90° triangle which implies that ∠B = 60° and ∠C = 30°

Using the Triangle Sum Theorem, we can conclude that ∠ADB = 90°, ∠ADE = 90°, ∠ AED = 60°, AND ∠ AEC = 120°

We can see that ΔAEC is an isosceles triangle. Draw a perpendicular to divide it into two congruent right triangles. Label the intersection as Z. ΔAEZ and ΔCEZ are 30°-60°-90° triangles.

Using the 30°-60°-90° rules for ΔABC we can calculate that AC = 5√3.

Since we divided ΔAEC into two congruent triangles, then AZ = [tex]\dfrac{5\sqrt 3}{2}[/tex]

Now use the 30°-60°-90° rules to calculate AE = 5

Find the coefficient of third term of (2x−1)^6.

Answers

240

using pascals trianle

for the power 6 it is

1, 6,15,20, 15,6, 1

and for the third term (2x)^4 and (-1)^2

[tex]15 \times {(2x)}^{4} \times {( - 1)}^{2} [/tex]

[tex]240 {x}^{4} [/tex]

Since only the coefficient is needed

the answer is 240.

The required coefficient of third term is 480.


Coefficient of the third term of (2x−1)^6 to be determine.

What is coefficient?

Coefficient is defined as the integer present adjacent to the variable.

Here,  (2x−1)^6
Using binomial expansion,
Third term = P(6,2)(2x)^6-2(-1)^2
                  =   6*5*16x^4
                  = 480x^4

Thus, the required coefficient of third term is 480.


Learn more about coefficient here:

https://brainly.com/question/2507029

#SPJ2

A cube whose edge is 20 cm 1 point
long, has circles on each of its
faces painted black. What is the
total area of the unpainted
surface of the cube if the
circles are of the largest
possible areas?(a) 90.72 cm2 (b)
256.72 cm² (c) 330.3 cm² (d)
514.28 cm?

Answers

Answer:

Unpainted  surface area = 514.28 cm²

Step-by-step explanation:

Given:

Side of cube = 20 Cm

Radius of circle = 20 / 2 = 10 Cm

Find:

Unpainted  surface area

Computation:

Unpainted  surface area = Surface area of cube - 6(Area of circle)

Unpainted  surface area = 6a² - 6[πr²]

Unpainted  surface area = 6[a² - πr²]

Unpainted  surface area = 6[20² - π10²]

Unpainted  surface area = 6[400 - 314.285714]

Unpainted  surface area = 514.28 cm²

Set A={XIX is an even whole number between 0 and 2) = 0
True? or false?​

Answers

false

Step-by-step explanation:

false

Drag each tile to the correct box.

Answers

Answer:

The order is 4) → 5) → 6) → 7) → 2) → 1) → 3)

Please find diagram with the arrangements

Step-by-step explanation:

The horizontal width of an hyperbola

For

1) [tex]\dfrac{(y - 11)^2}{7^2} -\dfrac{(x - 2)^2}{6^2} = 1[/tex]

h = 2, k = 11

The widths are;

Horizontal (h - a, k) to (h + a, k) which is (2 - 7, 11) to (2 + 7, 11) = 14 units wide

(h, k - b) to (h, k + b) which is (2, 11 -6) to (2, 11 + 6) = 12 units wide

2)

[tex]\dfrac{(y - 1)^2}{5^2} -\dfrac{(x - 7)^2}{12^2} = 1[/tex]

h = 7, k = 1

(h - a, k) to (h + a, k) which is (7 - 5, 1) to (7 + 5, 1) = Horizontal width 10 units wide

(h, k - b) to (h, k + b) which is (7, 1 -12) to (7, 1 + 12) = 24 units wide

3) [tex]\dfrac{(x - 6)^2}{6^2} -\dfrac{(y + 1)^2}{3^2} = 1[/tex]

h = 6, k = -1

a = 8, b = 3

The widths are;

(6 - 8, -1) to (6 + 8, -1) Horizontal width = 16

(6, -1 - 3) to 6, -1 + 3) width = 6

4) [tex]\dfrac{(x - 4)^2}{2^2} -\dfrac{(y + 2)^2}{5^2} = 1[/tex]

h = 4, k = -2, a = 2, b = 5

(4 - 2, (-2)) to (4 + 2, (-2)) Horizontal width = 4

(4, -2 - 5) to (4, -2 + 5) width = 10

5) [tex]\dfrac{(y + 5)^2}{2^2} -\dfrac{(x + 4)^2}{3^2} = 1[/tex]

h = -4, k = -5, a = 2, b = 3

(-4 - 2, (-5)) to (-4 + 2, (-5)) Horizontal width = 4

(-4, -5 - 3) to (4, -5 + 3) width = 6

6) [tex]\dfrac{(y + 1)^2}{2^2} -\dfrac{(x - 1)^2}{9^2} = 1[/tex]

h = 1, k = -1, a = 2, b = 9

(1 - 2, (-1)) to (1 + 2, (-1)) Horizontal width = 4

(1, -1 - 9) to (1, -1 + 9) width = 18

7) [tex]\dfrac{(x + 7)^2}{4^2} -\dfrac{(y - 9)^2}{9^2} = 1[/tex]

h = -7, k = 9, a = 4, b = 9

(-7 - 4, 9) to (-7 + 4, 9) Horizontal width = 8

(-7,  9 -9) to (-7, 9 + 9) width = 18

A cyclist travels at $20$ kilometers per hour when cycling uphill, $24$ kilometers per hour when cycling on flat ground, and $30$ kilometers per hour when cycling downhill. On a sunny day, they cycle the hilly road from Aopslandia to Beast Island before turning around and cycling back to Aopslandia. What was their average speed during the entire round trip?

Answers

Answer:

Average speed during the trip = 24 km/h

Step-by-step explanation:

Given:

Speed of cyclist uphill, [tex]v_1[/tex] = 20 km/hr

Speed of cyclist on flat ground = 24 km/h

Speed of cyclist downhill, [tex]v_2[/tex] = 30 km/h

Cyclist has traveled on the hilly road to Beast Island from Aopslandia and then back to Aopslandia.

That means, one side the cyclist went uphill will the speed of 20 km/h and then came downhill with the speed of 30 km/h

To find:

Average speed during the entire trip = ?

Solution:

Let the distance between Beast Island and Aopslandia = D km

Let the time taken to reach Beast Island from Aopslandia = [tex]T_1\ hours[/tex]

Formula for speed is given as:

[tex]Speed = \dfrac{Distance}{Time}[/tex]

[tex]v_1 = 20 = \dfrac{D}{T_1}[/tex]

[tex]\Rightarrow T_1 = \dfrac{D}{20} ..... (1)[/tex]

Let the time taken to reach Aopslandia back from Beast Island = [tex]T_2\ hours[/tex]

Formula for speed is given as:

[tex]Speed = \dfrac{Distance}{Time}[/tex]

[tex]v_2 = 30 = \dfrac{D}{T_2}[/tex]

[tex]\Rightarrow T_2 = \dfrac{D}{30} ..... (2)[/tex]

Formula for average speed is given as:

[tex]\text{Average Speed} = \dfrac{\text{Total Distance}}{\text{Total Time Taken}}[/tex]

Here total distance = D + D = 2D km

Total Time is [tex]T_1+T_2[/tex] hours.

Putting the values in the formula and using equations (1) and (2):

[tex]\text{Average Speed} = \dfrac{2D}{T_1+T_2}}\\\Rightarrow \text{Average Speed} = \dfrac{2D}{\dfrac{D}{20}+\dfrac{D}{30}}}\\\Rightarrow \text{Average Speed} = \dfrac{2D}{\dfrac{30D+20D}{20\times 30}}\\\Rightarrow \text{Average Speed} = \dfrac{2D\times 20 \times 30}{{30D+20D}}\\\Rightarrow \text{Average Speed} = \dfrac{1200}{{50}}\\\Rightarrow \bold{\text{Average Speed} = 24\ km/hr}[/tex]

So, Average speed during the trip = 24 km/h

Solve using quadratic formula.

1.)5x^2+13x=6

2.)3x^2+1=-5x

PLEASE HELP!!! WILL MARK BRAINLIEST!!!

Answers

Answer:

1. 2/5,-3 2. [tex]x=\frac{-5+-\sqrt{13} }{6}[/tex]

Step-by-step explanation:

i used the quadratic formula to find x also please note that 2 has 2 answers bc of the +- beofre the sqrt of 13  

Step-by-step explanation:

1).

5x² + 13x - 6 = 0

Using the quadratic formula

[tex]x = \frac{ - b± \sqrt{ {b}^{2} - 4ac} }{2a} [/tex]

a = 5 , b = 13 c = - 6

We have

[tex]x = \frac{ - 13± \sqrt{ {13}^{2} - 4(5)( - 6) } }{2(5)} [/tex]

[tex]x = \frac{ - 13± \sqrt{169 + 120} }{10} [/tex]

[tex]x = \frac{ - 13± \sqrt{289} }{10} [/tex]

[tex]x = \frac{ - 13±17}{10} [/tex]

[tex]x = \frac{ - 13 + 17}{10} \: \: \: \: \: or \: \: \: \: x = \frac{ - 13 - 17}{10} [/tex]

x = 2/5 or x = - 3

2).

3x² + 5x + 1 = 0

a = 3 , b = 5 , c = 1

[tex]x = \frac{ -5 ± \sqrt{ {5}^{2} - 4(3)(1)} }{2(3)} [/tex]

[tex]x = \frac{ - 5± \sqrt{25 - 12} }{6} [/tex]

[tex]x = \frac{ - 5± \sqrt{13} }{6} [/tex]

[tex]x = \frac{ - 5 + \sqrt{13} }{6} \: \: \: \: or \: \: \: x = \frac{ - 5 - \sqrt{13} }{6} [/tex]

Hope this helps you

If we did not write the equation 5x=21, instead we wrote it 21=5x,
we would get a different solution.
O True
O False

Answers

True we would get a different solution

Answer:

Step-by-step explanation:

5x = 21 and 21 = 5x are identical relationships, and so the solution would be the same in both cases.  (Commutative Property:  order of addition/subtraction is immaterial)

Find the length of the base and the height and calculate the area

Answers

Answer:

44

Step-by-step explanation:

base = 3- -5 = 8

height = 8 - -3 = 11

1/2 bh

1/2(8)(11) = 44

jim buys a calculator that is marked 30% off. If he paid $35, what was the original price?

Answers

Answer:

x = 50

Step-by-step explanation:

Let x be the original price.

He got 30% off

The discount is .30x

Subtract this from the original price to get the price he paid

x - .30x = price he paid

.70x = price he paid

.70x = 35

Divide each side  by .7

.70x/.7 = 35/.7

x=50

The answer is- $45.05

PLEASE help me with this question! No nonsense answers please. This is really urgent.

Answers

Answer:

The third option: x= [tex]\frac{8}{3} \pi[/tex]

Step-by-step explanation:

Arc length formula=[tex]\frac{Central Angle}{360} * 2\pi r[/tex]

Arc length = [tex]\frac{120}{360} *2\pi (4)[/tex]

=[tex]\frac{8}{3}\pi[/tex]

A timeline. 27 B C E to 180 C E PAX ROMANA. 44 B C E The Roman Empire was founded. 80 C E The Colosseum was built. 121 C E Hadrian's Wall was built in England to keep out enemies. 306 C E Constantine became emperor.
How many years passed between the building of the Colosseum and the building of Hadrian’s Wall?

201
121
41
36

Answers

Answer:

the answer is 41

Step-by-step explanation:

C. 41

Step-by-step explanation:

A mother who is 35 years old has two sons, one of whom is twice as old as the other. In 3 years the sum of all their ages will be 59 years. How old are the boys at present ?

Answers

Answer:

son2: 5

son1: 10

Step-by-step explanation:

2x (son1) + x (son2) + 35 (mother) + 3 (years)*3 (people) = 59

3x = 15

x = 5

The age of each boy at present will be 2 years and 3 years.

What is the linear system?

A linear system is one in which the parameter in the equation has a degree of one. It might have one, two, or even more variables.

Let the age of the sons will be x and y.

A mother who is 35 years old has two sons, one of whom is twice as old as the other. Then the equation will be

x = 2y

In 3 years, the sum of all their ages will be 59 years. Then the equation will be

x + y + x + 1 + y + 1 + x + 2 + y + 2 + 35 = 59

Simplify the equation, we have

3x + 3y + 41 = 59

      6y + 3y = 59 – 41

              9y = 18

                y = 2

Then the value of x will be

x = 2y

x = 2(2)

x = 4

Thus, the age of each boy at present will be 2 years and 3 years.

More about the linear system link is given below.

https://brainly.com/question/20379472

#SPJ2

Solve this problem... Really urgent ​

Answers

Answer:

[tex] \boxed{\sf Time \ taken = 15 \ minutes} [/tex]

Given:

Initial speed (u) = 65 km/h

Final speed (v) = 85 km/h

Acceleration (a) = 80 km/h²

To Find:

Time taken for car to achieve a speed of 85 km/h in minutes

Step-by-step explanation:

[tex]\sf From \ equation \ of \ motion:[/tex]

[tex] \boxed{ \bold{v = u + at}}[/tex]

By substituting value of v, u & a we get:

[tex] \sf \implies 85 = 65 + 80t[/tex]

Substract 65 from both sides:

[tex] \sf \implies 85 - 65 = 65 - 65 + 80t[/tex]

[tex] \sf \implies 20 = 80t[/tex]

[tex] \sf \implies 80t = 20[/tex]

Dividing both sides by 80:

[tex] \sf \implies \frac{ \cancel{80}t}{ \cancel{80}} = \frac{20}{80} [/tex]

[tex] \sf \implies t = \frac{2 \cancel{0}}{8 \cancel{0}} [/tex]

[tex] \sf \implies t = \frac{ \cancel{2}}{ \cancel{2} \times 4} [/tex]

[tex] \sf \implies t = \frac{1}{4} \: h[/tex]

[tex] \sf \implies t = \frac{1}{4} \times 60 \: minutes[/tex]

[tex] \sf \implies t = 15 \: minutes[/tex]

So,

Time taken for car to achieve a speed of 85 km/h in minutes = 15 minutes

Al’s Produce Stand sells 6 ears of corn for $1.50. Barbara’s Produce Stand sells 13 ears of corn for $3.12. Write two equations, one for each produce stand, that model the relationship between the number of ears of corn sold and the cost.

Answers

Answer:

6n = 1.50

and

13n = 3.12

Step-by-step explanation:

Here in this question, we are interested in writing equations that relate the number of ears of corn sold and the cost.

For Al’s produce stand, let the price per corn sold be n

Thus;

6 * n = 1.50

6n = $1.50 •••••••(i)

For the second;

let the price per corn sold be n;

13 * n = $3.12

-> 13n = 3.12 •••••••••(ii)

Angles L and M are supplementary. What is the sum of
their measures?
The sum of the measures of angles L and M is

Answers

180 degree

Step-by-step explanation:

supplementary means anhke havinv sum of 180 degree

so sum to two supplemrntary angles is 180 drgree

Answer: 180

Supplementary angles always add to 180.

One way I think of it is "supplementary angles form a straight angle", and both the words "supplementary" and "straight" start with the letter "S".

In contrast, complementary angles form a corner. Both "complementary" and "corner" start with "co". By "corner", I mean a 90 degree corner.

Other Questions
For the quarter ended March 31, 2017, Croix Company accumulates the following sales data for its newest guitar, The Edge: $314,000 budget; $300,800 actual. In the second quarter, budgeted sales were $381,000, and actual sales were $391,000.Prepare a static budget report for the second quarter and for the year to date.CROIX COMPANYSales Budget ReportFor the Quarter Ended June 30, 2017Second Quarter Year to DateProduct Line Budget Actual Difference Budget Actual Difference Anyone want to help...? Find the derivative of the function f(x) = (x3 - 2x + 1)(x 3) using the product rule.then by distributing and make sure they are the same answer Which kind of research would best enable you to predict academic success on the basis of performance on standard intelligence tests Write 11 numbers in a row so that the sum of any 3 consecutive numbers is negative, while the sum of all the numbers is positive. 1. When you write a paper, stick to your point. Don't introduce any topics.A. extraneousB. nefariousC. didacticD. diminutive Identify the location of the Harappan civilization on this map of the Indian subcontinent. A glacier advances at 7.4 x 10^-6 cm/sHow far will it move in 4.2 y?Answer in units of cm.cm/s. Jason is traveling by car from his home to his office. He has gone 3.5 miles so far. From this point on, he can cover 100 miles every 2 hours. What is the equation of a line that models the total miles traveled, y, in x hours after this point? A. Matter is anything that has mass and takes upspace.Which of the following is an example ofmatter?A. ear phonesB. musicC. sunlightD. heat Duff Inc. paid a 2.34 dollar dividend today. If the dividend is expected to grow at a constant 1 percent rate and the required rate of return is 11 percent, what would you expect Duff's stock price to be 4 years from now? can a steam cell taken from a baby, and giving the cell all that needs, can make clone Sort the tiles to show the characteristics of life that they describe. 10. Which of the following is NOT a method of transmission for the HIV Virus? A. O Casual contact with an infected person B. O Unprotected Sex C. O Sharing IV drug needles D. O Coming in contact with blood or body fluids On July 9, Mifflin Company receives a $8,600, 90-day, 12% note from customer Payton Summers as payment on account. What entry should be made on July 9 to record receipt of the note [fill in the blank]In this figure,AB and CD are parallel.AB is perpendicular to line segment_____. If the length of EF is a units, then the length of GH is_____units. In a double-slit experiment, light from two monochromatic light sources passes through the same double slit. The light from the first light source has a wavelength of 587 nm. Two different interference patterns are observed. If the 10th order bright fringe from the first light source coincides with the 11th order bright fringe from the second light source, what is the wavelength of the light coming from the second monochromatic light source? Research conducted on prejudicial attitudes and discriminatory behaviors revealed that college students who expressed higher than average feelings of prejudice toward Arab-Americans were A line passes through A(3,7) and B(-4,9). Find the value of a if C(a, 1) is on the line. Describe a situation (real or fictional) where not having a focused message in your writing could lead to a problem or poor outcome. Describe how you can use the writing process to ensure your writing is focused and effective.